A firm's demand curve is given by Q = 100 – 0.67P. What is the firm's corresponding marginal revenue curve?

Answers

Answer 1

To find the firm's corresponding marginal revenue curve, we need to first understand that marginal revenue is the change in total revenue resulting from a one-unit change in output. Mathematically, it can be expressed as the derivative of total revenue with respect to quantity.

In this case, we can find the total revenue function by multiplying price (P) and quantity (Q). So, TR = P*Q. Substituting the demand function Q = 100 – 0.67P, we get TR = P*(100 – 0.67P) = 100P – 0.67P².

To find the marginal revenue, we take the derivative of the total revenue function with respect to Q. So, MR = d(TR)/dQ.

Differentiating TR = 100P – 0.67P² with respect to Q, we get MR = 100 – 1.34P.

Therefore, the firm's corresponding marginal revenue curve is MR = 100 – 1.34P.
Therefore, the firm's corresponding marginal revenue curve is MR = 100 – 1.34P.

To know more about marginal revenue curve, refer

https://brainly.com/question/17049363

#SPJ11


Related Questions

classify the statements as true or false. δh for an endothermic reaction is positive. answer δh for an exothermic reaction is positive. answer

Answers

Answer:The statement "δH for an endothermic reaction is positive" is true.

The statement "δH for an exothermic reaction is positive" is false.

Explanation: ΔH (delta H) represents the change in enthalpy of a reaction. For an endothermic reaction, energy is absorbed from the surroundings, resulting in an increase in the internal energy of the system, and therefore ΔH is positive. In contrast, for an exothermic reaction, energy is released to the surroundings, resulting in a decrease in the internal energy of the system, and therefore ΔH is negative.

learn more about endothermic reaction

https://brainly.com/question/28909381?referrer=searchResults

#SPJ11

A Saturn V Moon rocket has a mass at lift-
off of 3.0 x 106 kg. The thrust at lift-off is
3.3 × 107 N. Find:
a) the weight of the rocket on Earth
b) the resultant (unbalanced) force at lift-off
c) the acceleration at lift-off
d) the apparent weight of the rocket in orbit.

Answers

Explanation:

a) weight = m * g = 3 x 10^6 kg   * 10 m/s^2 = 3.0 x 10^7  N

b)    Thrust - weight = 3.3 x 10^7 N  - 3.0 x 10^7 N    = 3 x 10^6 N

c)  F = ma        3. x 10^6   =   3 X 10^6  * a     solve for 'a' = 1 m/s^2

d)  weightless  (but not massless)

The 10-kg semicircular disk is rotating at ω-4 rad/s at the instant θ 60°. Determine the normal and frictional forces it exerts on the ground at A at this instant. Assume the disk does not slip as it rolls

Answers

The normal force at A is 98.1 N, and the frictional force at A is 49.05 N.

To determine the normal and frictional forces at A, follow these steps:
1. Calculate the gravitational force acting on the disk: F_gravity = mass × g = 10 kg × 9.81 m/s² = 98.1 N.
2. Determine the vertical component of the gravitational force acting on point A: F_vertical = F_gravity × cos(θ) = 98.1 N × cos(60°) = 49.05 N.
3. Calculate the normal force at A: F_normal = F_gravity - F_vertical = 98.1 N - 49.05 N = 98.1 N (since the disk is in equilibrium).
4. Calculate the torque caused by friction: τ = I × α, where I is the moment of inertia and α is the angular acceleration. Since the disk does not slip, α = 0, so τ = 0.
5. As there's no net torque, the frictional force must be equal to the vertical component of the gravitational force: F_friction = F_vertical = 49.05 N.

Learn more about friction here:

https://brainly.com/question/13000653

#SPJ11

a rock attached to a string swings back and forth every 6.4 s. how long is the string?

Answers

The length of the string is approximately 10.36 meters.

To calculate the length of the string for a pendulum that swings back and forth every 6.4 seconds, we can use the formula for the period of a simple pendulum: T = 2π√(L/g), where T is the period, L is the length of the string, and g is the acceleration due to gravity (approximately 9.81 m/s²).

Given the period T = 6.4 s, we can rearrange the formula to solve for L:

L = (T² * g) / (4π²)

L = ((6.4 s)² * 9.81 m/s²) / (4π²)

L ≈ 10.36 m

The length of the string is approximately 10.36 meters.

To know more about string visit:-

https://brainly.com/question/4087119

#SPJ11

consider the following mos amplifier where r1 = 541 k, r2 = 425 k, rd= 45 k, rs = 21 k, and rl=100 k. the mosfet parameters are: kn = 0.41 ma/v, vt = 1v, and =0.0133 v-1. find the voltage gain

Answers

The voltage gain of the given MOS amplifier is -0.766 V/V.

Consider the given MOS amplifier with the given values of resistors and MOSFET parameters. To find the voltage gain, we need to first calculate the small-signal voltage gain using the formula Av=-gm*(rd||RL), where gm is the transconductance of the MOSFET and rd||RL is the parallel combination of the drain resistor rd and the load resistor RL.

To calculate the transconductance gm, we use the formula gm=2*kn*(W/L)*(Vgs-Vt), where kn is the MOSFET transconductance parameter, W/L is the ratio of the width to the length of the MOSFET channel, Vgs is the gate-to-source voltage, and Vt is the threshold voltage of the MOSFET.

Using the given values, we get gm=0.0198 mS. Now, to find rd||RL, we add the values of rd and RL in parallel, which gives us a value of 38.710 k. Substituting these values in the small-signal voltage gain formula, we get Av=-0.766 V/V.

To know more about the voltage gain, click here;

https://brainly.com/question/28891489

#SPJ11

A mirror is rotated at an angle of 10° from its original position. How much is the rotation of the angle of reflection from its original position?
a. 5°
b. 10°
c. 15°
d. 20°
e. 25°
f. 30°

Answers

The rotation of the angle of reflection from its original position is 20°.When a mirror is rotated at an angle.

Since the angle of incidence is equal to the angle of reflection, the angle of reflection also changes by 20° (twice the angle of rotation) from its original position. Therefore, the rotation of the angle of reflection from its original position is 20°. The rotation of the angle of reflection from its original position when a mirror is rotated at an angle of 10° is 20°.

According to the law of reflection, the angle of incidence is equal to the angle of reflection. When a mirror is rotated, both the angle of incidence and the angle of reflection change. If the mirror is rotated by 10°, the angle of incidence changes by 10°, and since the angle of reflection is equal to the angle of incidence, the angle of reflection also changes by 10°. Therefore, the total change in the angle of reflection is 10° + 10° = 20°.


To know more about reflection visit

https://brainly.com/question/30270479

#SPJ11

a ship is sending out a sonar pulse to the ocean floor. if the pulse suddenly takes longer to return to the ship, most likely there is

Answers

If the sonar pulse suddenly takes longer to return to the ship, it suggests that there is an increase in the distance between the ship and the ocean floor or an increase in the speed of sound in the water.

Here are a couple of possibilities:

1. The ship has moved farther away from the ocean floor: If the ship has moved to a greater distance from the ocean floor, it will take a longer time for the sonar pulse to travel to the bottom and back to the ship. This could occur if the ship is moving away from the location where the initial pulse was sent or if the ship is in motion and has increased its distance from the ocean floor.

2. There is a change in the speed of sound in water: The speed of sound in water can be affected by various factors such as temperature, salinity, and pressure. If any of these factors change, the speed of sound in water can also change. If the speed of sound in the water has increased, it will take a longer time for the sonar pulse to travel to the bottom and back to the ship, resulting in a longer return time.

To determine the exact cause of the longer return time, further investigation and analysis of the situation would be necessary.

To know more about sonar pulse here

https://brainly.com/question/32005904

#SPJ4

Two small slits are in a thick wall, 27.5 cm apart. A sound source from the behind the wall emits a sound wave toward the wall at a frequency of 2,000 Hz. Assume the speed of sound is 342 m/s. (a) Find the (positive) angle (in degrees) between the central maximum and next maximum of sound intensity. Measure the angle from the perpendicular bisector of the line between the slits. ° (b) The sound source is now replaced by a microwave antenna, emitting microwaves with a wavelength of 2.75 cm. What would the slit separation (in cm) have to be in order to give the same angle between central and next maximum of intensity as found in part (a)? cm (c) The microwave antenna is now replaced by a monochromatic light source. If the slit separation were 1.00 µm, what frequency (in THz) of light would give the same angle between the central and next maximum of light intensity?

Answers

(a) To find the angle between the central maximum and next maximum of sound intensity, we can use the equation d sin θ = (m + 1/2)λ, where d is the distance between the slits, θ is the angle between the perpendicular bisector and the line connecting the slits to the central maximum, m is the order of the maximum (0 for the central maximum, 1 for the first maximum, etc.), and λ is the wavelength of the sound wave. Rearranging the equation, we get sin θ = (m + 1/2)λ/d. Plugging in the values given, we get sin θ = (1 + 1/2)(0.0171)/0.275, which gives us θ = 23.7°.

(b) To find the slit separation for microwaves, we can use the same equation as in part (a), but with the wavelength of the microwaves and the angle we just found. Rearranging, we get d = (m + 1/2)λ/sin θ. Plugging in the values, we get d = (1 + 1/2)(0.0275)/sin 23.7°, which gives us d = 0.053 cm.
(c) To find the frequency of light that would give the same angle between the central and next maximum of intensity, we can use the equation d sin θ = mλ, where d is the slit separation, θ is the angle we just found, m is the order of the maximum (0 for the central maximum, 1 for the first maximum, etc.), and λ is the wavelength of the light. Rearranging, we get λ = d sin θ/m. Plugging in the values, we get λ = (1.00 × 10^-6) sin 23.7°/1, which gives us λ = 3.81 × 10^-7 m. Using the speed of light (3 × 10^8 m/s), we can find the frequency: f = c/λ = 7.87 × 10^14 Hz, or 787 THz.

To know more about wavelength visit:

https://brainly.com/question/31143857

#SPJ11

Two pulleys with different radii (labeled a and b) are attached to one another so that they rotate together. Each pulley has a string wrapped around it with a weight hanging from it. The pulleys are free to rotate about a horizontal axis through the center. The radius of the larger pulley is twice the radius of the smaller one (b = 2a). A student describing this arrangement states: "The larger mass is going to create a counterclockwise torque and the smaller mass will create a clockwise torque. The torque for each will be the weight times the radius, and since the radius for the larger pulley is double the radius of the smaller, and the weight of the heavier mass is less than double the weight of the smaller one, the larger pulley is going to win. The net torque will be clockwise, and so the angular acceleration will be clockwise." What, if anything, is wrong with this contention? If something is wrong, explain how to correct it. If this contention is correct, explain why.

Answers

The contention made by the student is incorrect. While it is true that the torque for each weight is equal to the weight times the radius of the pulley, the calculation of net torque and direction of angular acceleration is incorrect.

How to explain the information

It's important to note that torque is a vector quantity, meaning that it has both a magnitude and direction. In this case, the torque created by each weight is in opposite directions (clockwise for the smaller weight and counterclockwise for the larger weight), so they cannot simply be added together to get a net torque.

The weight of the heavier mass is not less than double the weight of the smaller one, as the student claims. The weight of an object is proportional to its mass, and assuming both weights are located at the same distance from the center of rotation, the torque created by each weight is proportional to its weight.

Learn more about torque on

https://brainly.com/question/17512177

#SPJ4

A string 1.5 m long with a mass of 2.1 g is stretched between two fixed points with a tension of 95 N.
Find the frequency of the fundamental on this string.
Express your answer using two significant figures.

Answers

The fundamental on this string has a frequency of roughly 49.4 Hz.

To solve this problem

The following formula can be used to determine a wave's speed on a string:

v = sqrt(T/μ)

where T is the string's tension and is the string's linear mass density (mass per unit length). By dividing the string's mass by its length, we may calculate :

μ = m/L = 2.1 g / 1.5 m = 1.4 g/m = 0.0014 kg/m

Substituting the values of T and μ into the formula for v, we get:

v = sqrt(95 N / 0.0014 kg/m) ≈ 148.3 m/s

The formula: can be used to determine the fundamental frequency on the string, or the lowest resonant frequency.

f = v / (2L)

where L is the length of the string. Substituting the values of v and L, we get:

f = 148.3 m/s / (2 × 1.5 m) ≈ 49.4 Hz

Therefore, The fundamental on this string has a frequency of roughly 49.4 Hz.

Learn more about wave's speed here : brainly.com/question/29798763

#SPJ1

what was the original far point of a patient who had laser vision correction to reduce the minimum power of her eye by 4.75 diopters, producing normal distant vision for her? assume a distance from the eye lens to the retina of 2.00 cm, so the minimum power for normal vision is 50.0 diopters.

Answers

The original far point of the patient is 2.2cm

What is power of a lens?

The power of a lens is defined as the reciprocal of its focal length. It is represented by the letter P.

The power P of a lens of focal length f (in m) is given by. P=1/f. The SI unit of power of a lens is 'dioptre'.

If the minimum power for normal vision is 50diopters

Then the focal length of the eye lens = 1/50 = 0.02m

If the minimum power of the patient is reduced by 4.75

= 50-4.75 = 45.25 diopters

the original focal length = 1/45.25

= 0.022m = 2.2 cm

learn more about power of lens from

https://brainly.com/question/30995178

#SPJ4

astronomers use two points in earth’s orbit to get the best possible parallax measurement. even better measurements would be possible with observations from

Answers

Astronomers use two points in Earth's orbit, six months apart, to obtain the best possible parallax measurement.

Even better measurements would be possible with observations from multiple points in Earth's orbit, allowing for a more comprehensive and accurate assessment of parallax. By obtaining observations at different times and locations around the Sun, astronomers can minimize errors and enhance the precision of parallax measurements. This would lead to more precise determinations of distances to celestial objects and a deeper understanding of their spatial relationships within the universe. Astronomers use two points in Earth's orbit, six months apart, to obtain the best possible parallax measurement.

learn more about astronomers here:

https://brainly.com/question/1764951

#SPJ11

true/false. the centroidal axis and neutral axis are always the same in both straight and curved beam

Answers

The statement " The centroidal axis and neutral axis are always the same in both straight and curved beam" is false.

In straight beams, the centroidal axis and neutral axis are coincident because the cross-section of a straight beam is symmetric about the centroidal axis. However, in curved beams, the centroidal axis and neutral axis may not coincide because the cross-sectional area of a curved beam is not symmetric about the centroidal axis.

The neutral axis of a curved beam is the axis passing through the centroid of the cross-sectional area that is subjected to zero stress when the beam is loaded. In general, the neutral axis of a curved beam is located at a distance from the centroidal axis that depends on the curvature of the beam and the shape of the cross-section.

To know more about the beams refer here :

https://brainly.com/question/3132489#

#SPJ11

238U decays spontaneously by α emission to 234Th. The atomic masses are 238.050788 u for 238U and 234.043601 u for 234Th.
A. Calculate the total energy released by this process.
B. Calculate the recoil velocity of the 234Th nucleus.

Answers

A. The total energy released by this process is 4.27 MeV.

B. The recoil velocity of the 234Th nucleus is 2.05 x 10⁵ m/s.

A. The total energy released in this process can be calculated using the mass-energy equivalence formula

E=Δmc²,

where Δm is the mass difference between the initial and final states and c is the speed of light.

Δm = 238.050788 u - 234.043601 u

Δm = 4.007187 u

Converting the mass difference to energy using the conversion factor of 1 u = 931.5 MeV/c²,

ΔE = Δm * 931.5 MeV/c²

ΔE = 4.007187 u × 931.5 MeV/c²

ΔE = 3.73 MeV (rounded off to two significant figures)

Adding the energy released as kinetic energy of the α-particle, which has a kinetic energy of 0.54 MeV, the total energy released is

Total energy released = 3.73 MeV + 0.54 MeV

Total energy released = 4.27 MeV

B. The recoil velocity of the 234Th nucleus can be calculated using the conservation of momentum. Assuming that the α-particle is initially at rest and the recoiling 234Th nucleus has a mass of m and velocity v, the conservation of momentum can be written as

0 = mαvα + m×v

where mα and vα are the mass and velocity of the α-particle. Rearranging the equation, we get

v = - mα/m × vα

The mass of the α-particle is 4.001506 u and its kinetic energy is 0.54 MeV, which can be converted to momentum using the formula p = √(2mK), where K is the kinetic energy.

pα = √(2 × 4.001506 u × 0.54 MeV) / c

pα = 2.32 x 10⁻²² kg m/s

Substituting the values, we get

v = - (4.001506 u / 234.043601 u) × (2.32 x 10⁻²² kg m/s)

v = - 2.05 x 10⁵ m/s (rounded off to two significant figures)

The negative sign indicates that the 234Th nucleus recoils in the opposite direction to the α-particle.

To learn more about recoil velocity, here

https://brainly.com/question/14232084

#SPJ4

(a) If the planes of a crystal are 3.50Å (1Å= 10E-10 = Ångstrom unit) apart, what wavelength of electromagnetic waves are needed so that the first strong interference maximum in the Bragg reflection occurs when the waves strike the planes at an angle of 15.0 degrees?
(a2) In what part of the electromagnetic spectrum do these waves lie?
(a3) At what other angles will strong interference maxima occur?

Answers

a)The wavelength of the electromagnetic waves needed for the first strong interference maximum in the Bragg reflection is 1.05 Å.

a2) Electromagnetic spectrum do these waves lie in  X-ray part .

a3) The second strong interference maximum occurs at an angle of 9.0°. We can repeat this process to find the angles for other maxima.

(a) The Bragg's law relates the wavelength of X-rays to the spacing between the crystal planes and the angle at which the X-rays are incident on the crystal:

nλ = 2d sinθ

where n is an integer representing the order of the diffraction peak, λ is the wavelength of the incident radiation, d is the spacing between the planes, and θ is the angle between the incident X-ray beam and the crystal planes.

In this case, we want to find the wavelength of the electromagnetic waves that give the first strong interference maximum, which corresponds to n=1. The spacing between the planes is given as d = 3.50 Å. The angle of incidence is θ = 15.0 degrees. So we can rearrange the Bragg's law to solve for λ:

λ = 2d sinθ / n = 2(3.50 Å) sin(15.0°) / 1

λ = 1.05 Å

Therefore, the wavelength of the electromagnetic waves needed for the first strong interference maximum in the Bragg reflection is 1.05 Å.

(a2) The wavelength of 1.05 Å corresponds to X-rays, which lie in the X-ray part of the electromagnetic spectrum.

(a3) The other strong interference maxima will occur at angles that satisfy the Bragg's law, i.e.,

nλ = 2d sinθ

For the first maximum (n=1), we found that θ = 15.0°. For higher maxima, we need to find the angles that satisfy this equation for larger values of n. For example, for n=2:

2λ = 2d sinθ

sinθ = λ / 2d = 1.05 Å / (2 × 3.50 Å) = 0.150

θ = sin⁻¹(0.150) = 9.0°

So the second strong interference maximum occurs at an angle of 9.0°. We can repeat this process to find the angles for other maxima.

Know more about  electromagnetic waves   here:

https://brainly.com/question/24319848

#SPJ11

How much current is flowing through a 55 watt light bulb that runs on


a 110 volt circuit? *



0. 5 amps



0. 5 watts



2 amps



6050 amps

Answers

The current flowing through the 55 watt light bulb is approximately 0.5 amps.

To calculate the current flowing through the light bulb, we can use Ohm’s law, which states that the current (I) flowing through a circuit is equal to the voltage (V) divided by the resistance ®. In this case, we are given the power (P) of the light bulb, which is 55 watts, and the voltage (V) of the circuit, which is 110 volts. Since power is equal to the product of voltage and current (P = V * I), we can rearrange the equation to solve for the current:

I = P / V

Substituting the given values, we have:

I = 55 watts / 110 volts

I ≈ 0.5 amps

Therefore, the current flowing through the 55 watt light bulb is approximately 0.5 amps.

It’s important to note that the power rating of a light bulb (in watts) indicates the rate at which it consumes electrical energy, while the current (in amps) represents the rate at which the electric charge flows through the circuit. In this case, the power rating is used to calculate the current flowing through the light bulb.

Learn more about Ohm’s law here:

https://brainly.com/question/1247379

#SPJ11

A beam of electrons moves at right angles to a 3.0 ✕ 10-2-t magnetic field. the electrons have a velocity of 2.5 ✕ 106 m/s. what is the magnitude of the forces on each electron?

Answers

The magnitude of the force on each electron in the magnetic field is 1.68 x 10^-17 N.

To find the force on each electron, we can use the formula F = qvBsinθ, where F is the force, q is the charge of an electron, v is the velocity of the electron, B is the magnetic field, and θ is the angle between the velocity and magnetic field. Given that the angle is 90° (right angles), sin90° = 1.

1. The charge of an electron (q) = -1.6 x 10^-19 C
2. The velocity of the electron (v) = 2.5 x 10^6 m/s
3. The magnetic field (B) = 3.0 x 10^-2 T

Now, plug these values into the formula: F = (-1.6 x 10^-19 C) x (2.5 x 10^6 m/s) x (3.0 x 10^-2 T) x sin(90°)
F = (-1.6 x 10^-19 C) x (2.5 x 10^6 m/s) x (3.0 x 10^-2 T) x 1
F ≈ -1.68 x 10^-17 N
Since we're asked for the magnitude, we take the absolute value, which is 1.68 x 10^-17 N.

Learn more about magnetic field here:

https://brainly.com/question/23096032

#SPJ11

A 1000-kg car travels at 22 m/s and then quickly stops in 3.8 s to avoid an obstacle. What is the initial speed of the car in mph? mph Submit Answer Tries 0/2 What is the initial kinetic energy of the car in kilojoules (kJ)? Submit Answer Tries 0/2 What is the initial momentum of the car? kg*m/s Submit Answer Tries 0/2 What is the magnitude of the impulse necessary to stop the car? kg*m/s Submit Answer Tries 0/2 What is the magnitude of the average force in kiloNewtons (kN) that stopped the car? kN Submit Answer Tries 0/2 What is the magnitude of the average acceleration that stopped the car? m/s2

Answers

The magnitude of the average acceleration that stopped the car can be calculated using the formula a = ∆v/∆t, where ∆v is the change in velocity and ∆t is the time taken to stop the car. Plugging in the values, we get a = -22/3.8 = -5.79 m/s^2 (the negative sign indicates deceleration).

The initial speed of the 1000-kg car in mph can be found by converting 22 m/s to mph, which is approximately 49.2 mph. The initial kinetic energy of the car can be calculated using the formula KE = 0.5*m*v^2, where m is the mass of the car and v is its velocity. Plugging in the values, we get KE = 0.5*1000*(22^2) = 242000 kJ.
The initial momentum of the car can be calculated using the formula p = m*v, where m is the mass of the car and v is its velocity. Plugging in the values, we get p = 1000*22 = 22000 kg*m/s. The magnitude of the impulse necessary to stop the car can be calculated using the formula J = ∆p, where ∆p is the change in momentum. Since the car comes to a complete stop, the change in momentum is simply the initial momentum, which is 22000 kg*m/s.

Therefore, the magnitude of the impulse is also 22000 kg*m/s. The magnitude of the average force in kiloNewtons (kN) that stopped the car can be calculated using the formula F = ∆p/∆t, where ∆p is the change in momentum and ∆t is the time taken to stop the car. Plugging in the values, we get F = 22000/3.8 = 5789.5 N = 5.7895 kN.

To know more about acceleration visit:

https://brainly.com/question/30660316

#SPJ11

which of the following is required to solve for the nonstandard cell potential using the nernst equation? select all that apply.

Answers

Therefore, the required factors to solve for the nonstandard cell potential using the Nernst equation are the standard cell potential, temperature, and concentrations of the species involved.

To solve for the nonstandard cell potential using the Nernst equation, the following factors are required:

Standard cell potential (E°): The standard reduction potential of the half-reactions involved in the cell reaction is needed. It provides a reference point for the calculation.

Temperature (T): The temperature at which the cell operates is required because the Nernst equation includes a term for temperature dependence.

Concentrations of species involved: The concentrations of the species participating in the cell reaction are necessary to calculate the nonstandard cell potential. The Nernst equation incorporates the logarithm of the concentration ratio.

Learn more about Nernst equation here:

https://brainly.com/question/32075126

#SPJ11

An ideal Otto cycle with a specified compression ratio is executed using (a) air, (b) argon, and (c) ethane as the working fluid. For which case will the thermal efficiency be the highest? Why?

Answers

For a given compression ratio, the thermal efficiency of the Otto cycle will be highest when the working fluid has the highest ratio of specific heats. In this case, argon has the highest ratio of specific heats and therefore it will give the highest thermal efficiency.

The thermal efficiency of an Otto cycle is given by:

η = 1 - (1/r)^(γ-1)

where r is the compression ratio and γ is the ratio of specific heats.

The thermal efficiency depends only on the compression ratio and the ratio of specific heats of the working fluid. Therefore, the working fluid itself does not affect the thermal efficiency. However, the ratio of specific heats is different for each of the three fluids:

For air, γ = 1.4

For argon, γ = 1.67

For ethane, γ = 1.25

For more question on thermal efficiency click on

https://brainly.com/question/24244642

#SPJ11

The thermal efficiency will be highest for ethane as the working fluid.

The thermal efficiency of the ideal Otto cycle is given by:

η = 1 - (1/r)^(γ-1)

where r is the compression ratio and γ is the ratio of specific heats for the working fluid.

For a given compression ratio, the thermal efficiency of the Otto cycle depends on the value of γ, which is different for different working fluids.

For air, γ = 1.4

For argon, γ = 1.67

For ethane, γ = 1.22

Using these values, we can calculate the thermal efficiency for each case and compare them.

Assuming the same compression ratio for all cases, the thermal efficiencies are:

η_air = [tex]1 - (1/r)^(0.4)[/tex]

η_argon =[tex]1 - (1/r)^{(0.67)[/tex]

η_ethane = [tex]1 - (1/r)^{(0.22)[/tex]

To determine which working fluid will give the highest thermal efficiency, we need to compare these values.

Since the exponent in the expression for thermal efficiency is smaller for ethane, it means that it has a higher thermal efficiency than air and argon for the same compression ratio.

Learn more about thermal efficiency here:

https://brainly.com/question/13039990

#SPJ11

Click the reset button.A.Series CircuitsBuild a simple series circuit that consists of 6 pieces of wire, 1 lightbulb, and 1 battery (voltage source).

Answers

A series circuit is a simple circuit that consists of one path for the current to flow through.

What is a series circuit, and how does it work?

According to the Ohm's Law, A series circuit is a type of circuit where the components are connected in a line, one after the other. In this type of circuit, the current flows through each component in sequence, meaning that the current passing through each component is the same.

This is because there is only one path for the current to flow through, and the resistance of each component adds up to create a total resistance for the circuit.

In a series circuit, if one component fails, the entire circuit will fail. This is because the current is unable to flow past the failed component, and the circuit becomes open. Additionally, the voltage is divided across each component in the circuit, meaning that the voltage across each component is proportional to its resistance.

Learn more about Ohm's Law

brainly.com/question/1247379

#SPJ11

shows the viewing screen in a double-slit experiment with monochromatic light. Fringe C is the central maximum a. What will happen to the fringe spacing if the wavelength of the light is decreased? b. What will happen to the fringe spacing if the spacing between the slits is decreased? c. What will happen to the fringe spacing if the distance to the screen is decreased? d. Suppose the wavelength of the light is 500 nm. How much farther is it from the dot on the screen in the center of fringe E to the left slit than it is from the dot to the right slit?

Answers

The fringe spacing in a double-slit experiment decreases as the wavelength of the light decreases, the spacing between the slits decreases, and the distance to the screen decreases. The difference in path length between the dot on the screen in the center of fringe E and the left slit is (3λd)/(2θ).

a. If the wavelength of the light is decreased, the fringe spacing will decrease. This is because fringe spacing is directly proportional to the wavelength of light.

b. If the spacing between the slits is decreased, the fringe spacing will increase. This is because fringe spacing is inversely proportional to the slit spacing.

c. If the distance to the screen is decreased, the fringe spacing will increase. This is because fringe spacing is inversely proportional to the distance between the slits and the screen.

d. Using the small angle approximation, the path difference between the dot in the center of fringe E and the left slit is approximately (d/2)sin(θ). The path difference to the right slit is the same but with the opposite sign for θ. The difference in path length is approximately d sin(θ) which equals 3λ/2. Assuming sin(θ) ≈ θ, the distance to the left slit is (3λd)/(2θ).

To know more about the double-slit experiment refer here :

https://brainly.com/question/30727105#

#SPJ11

a radio station broadcasts with a power of 90.13 kw. how many photons are produced each second if that station broadcasts at a frequency of 101.2 m hz

Answers

The radio station produces approximately 5.6 x [tex]10^2^4[/tex] photons every second at a frequency of 101.2 MHz with a power of 90.13 kW.

What is the estimated number of photons produced per second?

The number of photons produced by a radio station is determined by its power output and frequency. The formula used to calculate the number of photons produced per second is given by the equation:

n = (P/E) x Avogadro's number

Where n is the number of photons, P is the power in watts, E is the energy per photon (Planck's constant x frequency), and Avogadro's number is the number of particles per mole (6.022 x [tex]10^2^3[/tex]).

Using the given values of power (90.13 kW) and frequency (101.2 MHz), we can calculate the energy per photon to be 1.24 x [tex]10^-^2^5[/tex] joules. Substituting these values into the equation, we get:

n = (90.13 x [tex]10^3[/tex] / 1.24 x [tex]10^-^2^5[/tex]) x 6.022 x [tex]10^2^3[/tex]

n = 5.6 x [tex]10^2^4[/tex] photons/second

Therefore, a radio station broadcasting with a power of 90.13 kW at a frequency of 101.2 MHz produces approximately 5.6 x [tex]10^2^4[/tex] photons per second.

Learn more about Avogadro's number

brainly.com/question/1445383

#SPJ11

An ideal gas at 20∘C consists of 2.2×1022 atoms. 3.6 J of thermal energy are removed from the gas. What is the new temperature in ∘C∘C?

Answers

The new temperature of the ideal gas after removing 3.6 J of thermal energy is approximately 12.1°C.

To calculate the new temperature, we'll use the formula for the change in internal energy of an ideal gas, which is ΔU = (3/2)nRΔT, where ΔU is the change in internal energy, n is the number of moles, R is the ideal gas constant, and ΔT is the change in temperature.

First, we need to determine the number of moles (n) from the given number of atoms (2.2 × 10²² atoms). Since 1 mole contains Avogadro's number (6.022 × 10²³) of atoms, we can find n by dividing the number of atoms by Avogadro's number:

n = (2.2 × 10²² atoms) / (6.022 × 10²³ atoms/mol) ≈ 0.0365 moles

Next, we need to find the change in internal energy (ΔU), which is -3.6 J since thermal energy is being removed from the gas.

Now, we can rearrange the formula ΔU = (3/2)nRΔT to solve for the change in temperature (ΔT):

ΔT = ΔU / [(3/2)nR] = -3.6 J / [(3/2)(0.0365 moles)(8.314 J/mol K)] ≈ -7.9°C

Since the initial temperature was 20°C, the new temperature is:

New Temperature = Initial Temperature + ΔT = 20°C -7.9°C ≈ 12.1°C.

To know more about the internal energy, click here;

https://brainly.com/question/14668303

#SPJ11

use the parallel axis theorem to get the total moment of inertia for a pendulum of length L with a ball of radius r.
I is the moment of inertia about an axis through the pivot, m is the mass of the ball, g is Earths gravitational constant, b is the distance from the pivot at the top of the string to the center of mass if the ball. The moment of inertia of the ball about an axis through the center of the ball is Iball=(2/5)mr^2

Answers

To use the parallel axis theorem to calculate the total moment of inertia for a pendulum with a ball, we need to consider the individual moments of inertia and their distances from the axis of rotation.

The moment of inertia of the ball about an axis through the center of the ball is given as Iball = (2/5)mr^2, where m is the mass of the ball and r is the radius of the ball.

The total moment of inertia for the pendulum is the sum of the moment of inertia of the ball and the moment of inertia about the axis through the pivot.

Using the parallel axis theorem, the moment of inertia about the pivot axis can be calculated as follows:

I = Iball + mb^2

Where I is the total moment of inertia, m is the mass of the ball, b is the distance from the pivot at the top of the string to the center of mass of the ball.

Therefore, the total moment of inertia for the pendulum is I = (2/5)mr^2 + mb^2.

This equation takes into account both the rotation of the ball about its own axis and the rotation of the pendulum as a whole about the pivot point.

Learn more about the **parallel axis theorem** and its applications in physics here:

https://brainly.com/question/30460015?referrer=searchResults

#SPJ11

An ideal gas is at a temperature of 320 K. What is the average translational kinetic energy of one of its molecules?A 9.2 x 10-24 B 1.4 x 10-23C cannot tell without knowing the molar mass D. 6.6x10-21

Answers

To calculate the average translational kinetic energy of a molecule in an ideal gas, we can use the equation:
E = (3/2) kT,,   E = 8.31 x 10^-21 J

where E is the average translational kinetic energy, k is the Boltzmann constant (1.38 x 10^-23 J/K), and T is the temperature in Kelvin.

Substituting the given temperature of 320 K into the equation, we get:

E = (3/2) x (1.38 x 10^-23 J/K) x (320 K)

E = 8.31 x 10^-21 J

Therefore, the correct answer is option D, 6.6 x 10^-21 J is closest to the calculated value. This means that the average translational kinetic energy of one molecule in the given ideal gas at 320 K is approximately 6.6 x 10^-21 J.
To calculate the average translational kinetic energy of a molecule in an ideal gas, we can use the following equation:

Average translational kinetic energy = (3/2) * k * T

where k is Boltzmann's constant (1.38 × 10⁻²³ J/K) and T is the temperature in Kelvin.

Given that the temperature T is 320 K, we can plug the values into the equation:

Average translational kinetic energy = (3/2) * (1.38 × 10⁻²³ J/K) * (320 K)

Now, we can calculate the result:

Average translational kinetic energy = (3/2) * (1.38 × 10⁻²³ J/K) * (320 K) ≈ 6.6 × 10⁻²¹ J

So, the average translational kinetic energy of one molecule in the ideal gas is approximately 6.6 × 10⁻²¹ J. Therefore, the correct answer is D. 6.6 × 10⁻²¹.

For more information on Boltzmann's constant visit:

brainly.com/question/30639301

#SPJ11

an electron moves with a speed of 5.30×106 m/s. for related problem-solving tips and strategies, you may want to view a video tutor solution of an electron-diffraction experiment.
part a what is its de broglie wavelength ?
part b
proton moves with the same speed. Determine its de Broglie wavelength ?

Answers

Part a: The de Broglie wavelength of the electron is 1.37 x 10^-10 meters.

Part b: The de Broglie wavelength of the proton with the same speed is 7.46 x 10^-8 meters.

Part A:

The de Broglie wavelength of an object with momentum p is given by the formula:

λ = h / p

where λ is the de Broglie wavelength, h is Planck's constant (6.626 x 10^-34 J*s), and p is the momentum of the object.

Since the electron has a mass of 9.109 x 10^-31 kg and a speed of 5.30 x 10^6 m/s, its momentum can be calculated as:

p = mv = (9.109 x 10^-31 kg) * (5.30 x 10^6 m/s) = 4.83 x 10^-24 kgm/s

Plugging this value of momentum into the de Broglie wavelength formula, we get:

λ = h / p = (6.626 x 10^-34 Js) / (4.83 x 10^-24 kgm/s) = 1.37 x 10^-10 m

Therefore, 1.37 x 10^-10 meters is the de Broglie wavelength of the electron.

Part B:

Following the same approach as above, the momentum of the proton with the same speed as the electron can be calculated as:

p = mv = (1.673 x 10^-27 kg) * (5.30 x 10^6 m/s) = 8.87 x 10^-21 kgm/s

Using this value in the de Broglie wavelength formula, we get:

λ = h / p = (6.626 x 10^-34 Js) / (8.87 x 10^-21 kgm/s) = 7.46 x 10^-8 m

Therefore, 7.46 x 10^-8 meters is the de Broglie wavelength of the proton.

For more such questions on de Broglie wavelength, click on:

https://brainly.com/question/30216495

#SPJ11

Part a: The de Broglie wavelength of the electron is 1.37 x 10^-10 meters.

Part b: The de Broglie wavelength of the proton with the same speed is 7.46 x 10^-8 meters.

Part A:

The de Broglie wavelength of an object with momentum p is given by the formula:

λ = h / p

where λ is the de Broglie wavelength, h is Planck's constant (6.626 x 10^-34 J*s), and p is the momentum of the object.

Since the electron has a mass of 9.109 x 10^-31 kg and a speed of 5.30 x 10^6 m/s, its momentum can be calculated as:

p = mv = (9.109 x 10^-31 kg) * (5.30 x 10^6 m/s) = 4.83 x 10^-24 kgm/s

Plugging this value of momentum into the de Broglie wavelength formula, we get:

λ = h / p = (6.626 x 10^-34 Js) / (4.83 x 10^-24 kgm/s) = 1.37 x 10^-10 m

Therefore, 1.37 x 10^-10 meters is the de Broglie wavelength of the electron.

Part B:

Following the same approach as above, the momentum of the proton with the same speed as the electron can be calculated as:

p = mv = (1.673 x 10^-27 kg) * (5.30 x 10^6 m/s) = 8.87 x 10^-21 kgm/s

Using this value in the de Broglie wavelength formula, we get:

λ = h / p = (6.626 x 10^-34 Js) / (8.87 x 10^-21 kgm/s) = 7.46 x 10^-8 m

Therefore, 7.46 x 10^-8 meters is the de Broglie wavelength of the proton.

For more such questions on de Broglie wavelength, click on:

brainly.com/question/30216495

#SPJ11

Air at 20oC C and I atm flows over a flat plate at 40 m/s. The plate is 80 cm long and is maintained at 60oC. Properties of air at 40oC are Pr = 0.7, K = 0.02733 W/mK, Cp=1.007 kJkgK μ=1.906×10−5kgm−sand rho=1.128kg/m3.
The avergae heat transfer coefficient is ___Use ¯¯¯¯¯¯¯¯Nu=Pr13(0.036 R0.8e−871).
A. 69 W/m2K
B. 62 W/m2K
C. 88 W/m2K
D. 54 W/m2K

Answers

The problem provides us with the following parameters: Air temperature: 20°C, Air velocity: 40 m/s, Plate length: 80 cm = 0.8 m, Plate temperature: 60°C, Properties of air at 40°C: Pr = 0.7, K = 0.02733 W/mK, Cp = 1.007 kJ/kgK.

To find the average heat transfer coefficient, we can use the following equation: h = q / ([tex]T_{plate}[/tex] - [tex]T_{air}[/tex]), where: h: average heat transfer coefficient, q: heat flux (W/m2), [tex]T_{plate}[/tex] : plate temperature (K), [tex]T_{air}: air temperature (K). To find q, we can use the following equation:q = hA([tex]T_{plate}[/tex] - [tex]T_{air}[/tex]), where: A: plate area ([tex]m^{2}[/tex]), To find A, we need to convert the plate length from cm to m: A = Lw = (0.8 m)(1 m) = 0.8 [tex]m^{2}[/tex]. Now we need to find the Nusselt number (Nu), which is given by the equation: Nu = (0.036 [tex]Re^{0.8}[/tex])[tex]Pr^{1/3}[/tex], where: Re: Reynolds number. To find Re, we need to calculate the air density and viscosity at 20°C: ρ = 1.292 kg/[tex]m^{3}[/tex] (from the ideal gas law), μ = 1.789×[tex]10^{-5}[/tex] kg/m.s (from Sutherland's law). Now we can calculate the Reynolds number: Re = (ρV L) / μ = (1.292 kg/m3)(40 m/s)(0.8 m) / (1.789×[tex]10^{-5}[/tex] kg/m.s) = 364,468. Substituting the values into the Nusselt number equation, we get: Nu = 156.85. Now we can calculate the average heat transfer coefficient: h = NuK/L = (156.85)(0.02733 W/mK) / (0.8 m) = 5.33 W/m2K. Finally, we can calculate the heat flux: q = hA([tex]T_{plate}[/tex] - [tex]T_{air}[/tex]) = (5.33 W/m2K)(0.8 m2)(60 - 20)K = 1702.4 W. Therefore, the average heat transfer coefficient is 5.33 W/m2K.

Learn more about heat here :

https://brainly.com/question/31065010

#SPJ11

The average heat transfer coefficient is 69 W/m²K (option a).

1. Calculate the Reynolds number using Re = rho * V * L / mu, where V is the velocity, L is the length of the plate, mu is the dynamic viscosity, and rho is the density of air at 20°C.

Re = (1.128 kg/m³) * (40 m/s) * (0.8 m) / (1.906×[tex]10^{-5[/tex] kg/m s)

Re = 1.495×[tex]10^6[/tex]

2. Calculate the Nusselt number using the given equation Nu = [tex]Pr^{(1/3)} * (0.036 * Re^{(0.8)[/tex] * exp(-8.71/Pr)).

Nu = 0.[tex]7^{(1/3)[/tex]* (0.036 * (1.495× [tex]10^6)^{(0.8)[/tex] * exp(-8.71/0.7))

Nu = 259.65

3. Calculate the average heat transfer coefficient using the equation h = Nu * k / L, where k is the thermal conductivity of air at 40°C.

h = (259.65) * (0.02733 W/mK) / (0.8 m)

h = 8.841 W/m²K

4. Convert the heat transfer coefficient to watts per square meter kelvin using the equation q = h * (T_surface - T_air), where T_surface is the temperature of the plate and T_air is the temperature of the air.

q = (8.841 W/m²K) * (60°C - 20°C)

q = 353.64 W/m²

5. Finally, calculate the average heat transfer coefficient using the equation h_avg = q / (A * delta_T), where A is the surface area of the plate and delta_T is the temperature difference between the plate and the air.

A = 0.8 m * 1 m = 0.8 m²

delta_T = 60°C - 20°C = 40°C

h_avg = (353.64 W/m²) / (0.8 m² * 40°C)

h_avg = 11.05 W/m²K

The average heat transfer coefficient is 11.05 W/m²K, which is not one of the answer choices.

6. Therefore, the correct answer is to round up the result from step 3 to the nearest option, giving us an answer of 69 W/m²K.

For more such questions on heat, click on:

https://brainly.com/question/934320

#SPJ11

A ray of light traveling in a block of glass refracts into benzene. The refractive index of benzene is 1.50. If the wavelength of the light in the benzene is 500 nm and the wavelength in the glass is 455 nm, what is the refractive index of the glass? (a) 1.00 (b) 1.36 (c) 1.65 (d) 2.00 (e) none of the above answers

Answers

The refractive index of the glass is 1.36. The answer is (b)

The refractive index of a material is the ratio of the speed of light in vacuum to the speed of light in the material.

Using Snell's law, the ratio of the sine of the angle of incidence to the sine of the angle of refraction can be expressed as the ratio of the refractive indices of the two materials.

Therefore, we can use this relationship to solve for the refractive index of the glass.

Let ng be the refractive index of the glass. Using the given information, we can write:

sinθ1/sinθ2 = ng/1.50 = λ1/λ2

where θ1 and θ2 are the angles of incidence and refraction, λ1 is the wavelength in the glass, and λ2 is the wavelength in benzene.

Solving for ng, we have:

ng = (1.50 × λ1) / λ2 = (1.50 × 455 nm) / 500 nm ≈ 1.36

Therefore, the answer is (b) 1.36.

To know more about Snell's law, refer here:

https://brainly.com/question/2273464#

#SPJ11

a texas railroad section was recently surveyed with rtk and found to be 1908v x 1902v. what would half that acreage be calculated out to?

Answers

A property parcel's acreage can be determined by multiplying its length by its width and dividing the result by 43,560, the number of square feet in an acre.

The entire acreage can be estimated using the following formula given that the Texas railroad segment is 1908 feet by 1902 feet:

1908 feet by 1902 feet divided by 43,560 feet per acre equals 83.063 acres.

We can just split this acreage by two to get half of it:

Half an acre is equal to 83.063% of an acre, or 41.5315 acres.

Therefore, 41.53 acres would be about half of the Texas railway section. It's important to note that this computation makes the assumption that the parcel is rectangular and has straight edges.

For more such questions on length

https://brainly.com/question/24581635

#SPJ11

The size of a property lot can be calculated by multiplying its width and length and then dividing the product by 43,560, which is the equivalent of one acre in square feet.

How to solve

If the Texas railroad segment measures 1908 feet by 1902 feet, the total area can be computed utilizing this equation.

83063 acres can be calculated by dividing an area of 1908 feet by 1902 feet by the conversion factor of 43,560 feet per acre.

We can easily divide this piece of land into two equal parts, obtaining half of it.

An area of 0. 5 acres can be expressed as 83. 063% of an entire acre or approximately 41. 5315

Hence, the Texas railroad section would comprise roughly twice the area of 41. 53 It should be emphasized that in this calculation, the parcel is assumed to have a rectangular shape and its edges are straight.

Read more about square area here:

https://brainly.com/question/11444061

#SPJ4

Other Questions
evaluate cydx xydy along the given path c from (0,0) to (5,1). a. the parabolic path x=5y2.b) The straight-line path.c) The polygonal path (0,0),(0,1),(5,1).d) Thecubic path x=5y3 counts 29 members and is part of a continuing political and military alliance that prefers diplomatic solutions to military force is called A graph shows the horizontal axis numbered 1 to 5 and the vertical axis numbered 1 to 5. Points and a line show a downward trend. Which is most likely the correlation coefficient for the set of data shown? 0. 83 0. 21 0. 21 0. 83. Schubert's songs number more than. 600. Schubert wrote compositions in every musical genre except - string quartetes - piano concertos - symphonies - operas. If r = 0.65, what does the coefficient of determination equal?A. 0.194B. 0.423C. 0.577D. 0.806 passwords used by the ppp daemon are stored in two files: /etc/ppp/pap-secrets and /etc/ppp/chap-secrets. in traditional farming cultures, who is typically responsible for the livestock and farmland? Match the term to the appropriate blanks to complete the sentence. Question 7 options: biodiversity species diversity genetic diversity extinction extirpation species 1. A _____ is a distinct type of organism, a set of individuals that uniquely share certain characteristics and can breed with one another and produce fertile offspring. The length of the curve y=sinx from x=0 to x=34 is given by(a) 3/40sinx dx if we run lm() to fit the model for weightgain4, using light as an explanatory model, how is error from the model calculated for each mouse? Under optimal conditions one E. coli cell can become two cells. A. every 2 to 3 minutes. B. every 20 to 30 minutes. C. every 2 to 3 hours. which of the following enables a program to perform one or more actions repeatedly as a loop as long as a certain condition is met? In a monophasic oral contraceptive, what happens to the level of estrogen in the doses taken over the length of a menstrual cycle?It shifts across three dosage levels.It remains at the same dosage level.It is delivered at two dosage phases.It decreases across the dosage cycle. For given number, give normal form, precision, and magnitude: 0503.070a. Normalized:b. Precision:c. Magnitude:d. Stored with precision 5:e. Absolute error with precision 5:f. Relative error with precision 5: Exercise 7You have read in your local newspaper that developers would like to build a zoo in your townHere are some comments on the topic from people living in the town:It would be fascinating to seewild animals close up.Animals should be intheir natural habitat -zoos are cruel.TotalWe can see wild animals on TV. Thereis no need for a zoo.Write an article for the newspaper, giving your views.Modern zoos are caringplaces with lots offor the animals.space Evaluate the factorial expression 20!/ 17!(3-1)! Choose the correct answer from the options below a. 190 b. 1368 c. 3420 d. 58140 the instability of xenon fluorides is due to its negative enthalpy of formation. true false The owners of this house want to knock down the wall between the kitchen and the family room. What expression represents the area of the new combined open space?Family RoomX?+ 10x + 24KitchenX2 + 7x + 12 if the pharmacist is unable to identify a specific rationale for a medication in the patients regimen, the medication-related problem is categorized as: unnecessary medication therapy. T/F? Repeat Quick Check 7.1 (page 338) but in line 14 of SubsetSum, always choose the smallest element of W. Diagram of recursive invocations: Which strategy (largest element as in the original Quick Check or smallest element as here) seems better?